(a) Calculate the focal length of the mirror formed by the shiny back of a spoon that has a 2.30 cm radius of curvature. (b) What is its power in diopters?

Answers

Answer 1

(a) The focal length of the mirror formed by the shiny back of a spoon that has a 2.30 cm radius of curvature is 1.15 cm and (b) The power is 86.96 diopters.

(a) The focal length of a spherical mirror is half of its radius of curvature, so the focal length of the mirror formed by the shiny back of a spoon with a 2.30 cm radius of curvature is:
focal length = radius of curvature / 2
focal length = 2.30 cm / 2
focal length = 1.15 cm

Therefore, the focal length of the mirror is 1.15 cm.

(b) The power of a spherical mirror in diopters is given by the formula:

power = 1 / focal length (in meters)

Since the focal length is in centimeters, we need to convert it to meters first:
focal length in meters = 1.15 cm / 100
focal length in meters = 0.0115 m

Now we can calculate the power in diopters:
power = 1 / focal length
power = 1 / 0.0115
power = 86.96 diopters

Therefore, the power of the mirror formed by the shiny back of a spoon with a 2.30 cm radius of curvature is 86.96 diopters.

Learn more about 'spherical mirror':

https://brainly.com/question/16188698

#SPJ11


Related Questions

Two kids take part in a tug of war on an icy playground (don't try this at home). There is zero friction between their shoes and the ground. Child A has a mass of 26 - kg and child B has a mass of 49 - kg. They are initially standing 11 m apart. A. How far from child A is their CM? B. Each child holds the end of a rope and child B pulls on the rope so that he moves toward child A. How far will child B have moved when he collides with child A?

Answers

55 CM
Explanation
26 divide
829, 23-1,24,IB9
0,^uO992
Add them. You get 55CM

Two kids of different masses take part in a tug of war with no friction. The distance of their center of mass can be calculated, and if child B pulls on the rope towards child A, the distance he will move before colliding with child A can also be calculated.

A. To find the center of mass (CM) of the system, we need to take into account both the masses and their distances from each other. The formula for the position of the CM is:

CM = (m1x1 + m2x2) / (m1 + m2)

where m1 and m2 are the masses, x1 and x2 are their distances from a chosen reference point.

In this case, let's take child A as the reference point, so x1 = 0 (since child A is at the origin), and x2 = 11 m. Then we have:

CM = (m1x1 + m2x2) / (m1 + m2)

= (26 kg * 0 + 49 kg * 11 m) / (26 kg + 49 kg)

= 7.6 m

Therefore, the center of mass of the system is located 7.6 m from child A.

B. As child B pulls on the rope, he will move towards child A, and their separation distance will decrease. At the same time, the center of mass of the system will move towards child B. Since there is no external force acting on the system, the position of the center of mass will not change.

Let's assume that child B moves a distance of x towards child A before they collide. Then the distance between child A and the CM of the system will be (11 - x), and the distance between child B and the CM will be x. Using the formula for the position of the CM, we can set up an equation:

CM = (m1x1 + m2x2) / (m1 + m2)

= ((26 kg) * 0 + (49 kg) * (11 - x)) / (26 kg + 49 kg)

= (539 - 49x) / 75

Since the CM does not move, this must be equal to the initial position of the CM, which we found to be 7.6 m from child A:

(539 - 49x) / 75 = 7.6

Solving for x, we get:

x = 6.4 m

Therefore, child B will have moved a distance of 6.4 m towards child A before they collide.

To know more about the center of mass refer here :

https://brainly.com/question/28996108#

#SPJ11

What is the intensity at a point on the circle at an angle of 4. 70 ∘ from the centerline?

Answers

The intensity at a point on the circle at an angle of 4.70 degrees from the centerline is 0.45 W/m.

To calculate the intensity at the desired point, we can use the equation for the electric field strength of a point source:

E = kQ / r²

where E is the electric field strength, k is Coulomb's constant, Q is the charge of the source, and r is the distance from the source.

Since the two transmitters are broadcasting in phase, we can treat them as a single source with double the charge. We can then use the equation for the intensity of an electromagnetic wave:

I = c * ε * E²

where I is the intensity, c is the speed of light, ε is the electric constant, and E is the electric field strength.

Plugging in the given values, we get:

Q = 2 * (1575.42 MHz * 2π) / c = 4.04 × 10⁻¹⁹ C

r = (several hundred meters) * sin(4.70 degrees) = 39.6 m

E = kQ / r² = 1.03 × 10⁻⁶ N/C

I = c * ε * E² = 0.45 W/m

Therefore, the intensity at a point on the circle at an angle of 4.70 degrees from the centerline is 0.45 W/m.

Learn more about centerline:

https://brainly.com/question/17034418

#SPJ4


Complete Question:

The GPS (Global Positioning System) satellites are approximately 5.18 m across and transmit two low-power signals, one of which is at 1575.42 MHz (in the UHF band). In a series of laboratory tests on the satellite, you put two 1575.42 MHz UHF transmitters at opposite ends of the satellite. These broadcast in phase uniformly in all directions. You measure the intensity at points on a circle that is several hundred meters in radius and centered on the satellite. You measure angles on this circle relative to a point that lies along the centerline of the satellite (that is, the perpendicular bisector of a line which extends from one transmitter to the other). At this point on the circle, the measured intensity is 2.00 W/m. What is the intensity at a point on the circle at an angle of 4. 70 ∘ from the centerline?

which signal has a continues change in amplitude and frequency ?

Answers

The signal that has a continuous change in amplitude and frequency is known as an analog signal. This type of signal is characterized by its ability to represent a range of values using a continuous waveform, as opposed to a digital signal which represents values discretely.


The amplitude of an analog signal refers to the height of the waveform at a given point in time, while the frequency refers to the number of cycles per second. In an analog signal, both the amplitude and frequency can change continuously, resulting in a signal that is constantly varying in both amplitude and frequency.
Examples of analog signals include sound waves, radio waves, and electrical signals. These signals are used in a wide variety of applications, from music and communication to industrial automation and medical imaging.
Overall, the continuous change in amplitude and frequency of analog signals allows for more precise and nuanced representation of data, making them an important tool in many fields.

learn more about amplitude Refer: https://brainly.com/question/29546637

#SPJ11

Electromagnets and solid permanent magnets will both attract iron. How are electromagnets different than permanent magnets?
O A. Electromagnets can be made of plastic. O B. Permanent magnets can be turned off. O C. Permanent magnets use a coil of wire. OD. Electromagnets can be turned off

Answers

Electromagnets are different than permanent magnets because electromagnets can be turned off while permanent magnets cannot.

This is because an electromagnet uses a current flowing through a wire coil to create a magnetic field, and this current can be turned on and off, allowing the magnetic field to be controlled.

In contrast, a permanent magnet is made of a material with inherent magnetic properties that cannot be turned off. While both types of magnets can attract iron, the ability to turn off an electromagnet makes it more versatile and useful in a variety of applications, such as in electric motors and MRI machines.

Learn more about the electromagnets at

https://brainly.com/question/17057080

#SPJ4

The question is -

Electromagnets and solid permanent magnets will both attract iron. How are electromagnets different than permanent magnets?

A. Electromagnets can be made of plastic.

B. Permanent magnets can be turned off.

C. Permanent magnets use a coil of wire.

D. Electromagnets can be turned off.

according to stefan's law (see section 3.4 in the textbook), how much more radiation - per square meter, say - is emitted by venus's surface at 730 k than is emitted by earth's surface at 300 k ? express your answer using two significant figures.

Answers

The surface of Venus emits about 58.2 times more radiation per square meter than the surface of Earth, assuming they both behave as black bodies.

Stefan's law states that the energy radiated per unit area per unit time, or the radiant emittance, of a black body is proportional to the fourth power of its absolute temperature. Mathematically, this can be written as:

E = [tex]σT^4[/tex]

where E is the radiant emittance, σ is the Stefan-Boltzmann constant ([tex]5.67 x 10^-8 W/m^2K^4[/tex]), and T is the absolute temperature.

Using this formula, we can calculate the ratio of the radiant emittance of Venus's surface at 730 K to that of Earth's surface at 300 K:

([tex]E_venus / E_earth) = (σT_venus^4 / σT_earth^4[/tex])

([tex]E_venus / E_earth) = (T_venus / T_earth[/tex])[tex]^4[/tex]

([tex]E_venus / E_earth) = (730 / 300)^4[/tex]

([tex]E_venus / E_earth) ≈ 58.2[/tex]

Therefore, the surface of Venus emits about 58.2 times more radiation per square meter than the surface of Earth, assuming they both behave as black bodies.

To know more about radiation, visit:

https://brainly.com/question/13934832#

#SPJ11

work done ona closed system consisting of 2 kg of water initially at 160 oc, 10 bar undergoes an internally reversible, isothermal expansion during which there is energy transfer by heat into the system of 2700 kj. determine the work done, in kj. the system is negative or positive

Answers

The work done on the closed system consisting of 2 kg of water initially at 160°C and 10 bar, undergoing an internally reversible, isothermal expansion with energy transfer by heat into the system of 2700 kJ, is positive and can be calculated as follows:

The given problem involves an isothermal process, which means the temperature of the system remains constant throughout the process. According to the first law of thermodynamics, for an isothermal process, the work done is equal to the heat transferred into the system.

Given:

Mass of water (m) = 2 kg

Initial temperature (T) = 160°C = (160 + 273.15) K = 433.15 K (converting to Kelvin)

Initial pressure (P) = 10 bar = 10 × 10⁵ Pa (converting to Pascal)

Heat transferred (Q) = 2700 kJ = 2700 × 10³ J (converting to Joules)

Since the process is isothermal, the work done (W) is equal to the heat transferred (Q) into the system, i.e., W = Q.

Substituting the given values, we get:

W = 2700 × 10³ J = 2700 kJ

So, the work done on the system is 2700 kJ, and it is positive as the heat is transferred into the system during the expansion process.

To know more about isothermal process refer here:

https://brainly.com/question/12023162#

#SPJ11

Question:
Why does a rubber band become warm when stretched repeatedly?
Answer
Rubber bands are made of polymeric molecules, or molecules consisting of chains of many subunits linked together. When a rubber band is stretched, it...

Answers

Answer:

When molecules, not just rubber molecules, but any molecules, form crystals, they give off heat. This is why the rubber band feels hot when its stretched. When you let go of the rubber band, the polymer molecules break out of those crystals. Whenever molecules break out of crystals, they absorb heat.

Explanation:

When a rubber band is stretched, its polymeric molecules, consisting of chains of many subunits, become elongated and aligned. This stretching process increases the entropy or disorder within the rubber band as it converts potential energy into kinetic energy.

The energy required for this deformation comes from the work done by the person stretching the rubber band.

As the rubber band is stretched repeatedly, the internal molecular friction generates heat. The kinetic energy from the rapid realignment of the polymer chains is converted into thermal energy, causing the rubber band to feel warm. This phenomenon is known as hysteresis heating, a result of the viscoelastic nature of rubber materials.

Viscoelastic materials exhibit both viscous and elastic properties when undergoing deformation. The elastic component allows the rubber band to return to its original shape after being stretched, while the viscous component dissipates some of the applied energy as heat, resisting the rapid change in shape.

In summary, when a rubber band is repeatedly stretched, its polymeric molecules experience an increase in entropy due to the conversion of potential energy into kinetic energy. This, combined with the viscoelastic properties of the material, generates internal molecular friction, ultimately causing the rubber band to warm up through a process known as hysteresis heating.

For more such questions on Stretching a rubber band.

https://brainly.com/question/30045748#

#SPJ11

the area of the floor of a room is 132m2 if the length of the room is 12 m find its bread​

Answers

The area of the floor of a room is 132 [tex]m^{2}[/tex] if the length of the room is 12 m.

To find the breadth of the room, we need to use the formula for the area of a rectangle, which is given as

Area = length × breadth

We are given the area of the room as 132 square meters, and the length of the room as 12 meters. We can rearrange the formula above to solve for the breadth.

Breadth = Area / length

Substituting the given values, we get

Breadth = 132 m² / 12 m

Breadth = 11 meters.

Hence, the breadth of the room is 11 meters.

To know more about area here

https://brainly.com/question/30811800

#SPJ1

The classic Goodyear blimp is essentially a helium balloon a big one, containing 5700 m3 of helium. If the envelope and gondola have a total mass of 4300kg, what is the maximum cargo load when the blimp flies at a sea-level location? Assume an air temperature of 20oC.

Answers

The maximum cargo load of the Goodyear blimp is 2568.8 kg when flying at a sea-level location with an air temperature of 20°C.

To solve this problem, we need to use Archimedes' principle, which states that the buoyant force on an object is equal to the weight of the fluid displaced by the object. In this case, the fluid is air, and the buoyant force on the blimp is equal to the weight of the air displaced by the blimp.

First, we need to calculate the weight of the blimp, which is equal to the sum of the envelope and gondola:

Weight of blimp = 4300 kg

Next, we need to calculate the weight of the air displaced by the blimp. We can use the density of air at 20°C, which is approximately 1.204 kg/m³:

Volume of blimp = 5700 m³

Weight of air displaced = Volume of blimp x Density of air = 5700 x 1.204 = 6868.8 kg

Finally, we can calculate the maximum cargo load by subtracting the weight of the blimp from the weight of the air displaced:

Maximum cargo load = Weight of air displaced - Weight of blimp = 6868.8 - 4300 = 2568.8 kg

To know more about helium balloon, here

brainly.com/question/12666223

#SPJ1

For each quantity listed, indicate dimensions using force as a primary dimension, and give typical SI and English units: a. Power b. Pressure c. Modulus of elasticity d. Angular velocity e. Energy f. Momentum g. Shear stress h. Specific heat i. Thermal expansion coefficient j. Angular momentum

Answers

When working with physical quantities, it is important to understand their dimensions and units of measurement. Understanding the dimensions and units of the quantities can be useful in a variety of scientific and engineering contexts, from designing machines to measuring the properties of materials.

The dimensions and typical units for each quantity:
a. Power:
Dimensions: Force × Length × Time^(-2)
SI units: Watts (W)
English units: Foot-pounds per second (ft·lb/s)

b. Pressure:
Dimensions: Force × Length^(-2)
SI units: Pascals (Pa)
English units: Pounds per square inch (psi)

c. Modulus of elasticity:
Dimensions: Force × Length^(-2)
SI units: Pascals (Pa)
English units: Pounds per square inch (psi)

d. Angular velocity:
Dimensions: Time^(-1)
SI units: Radians per second (rad/s)
English units: Revolutions per minute (rpm)

e. Energy:
Dimensions: Force × Length
SI units: Joules (J)
English units: Foot-pounds (ft·lb)

f. Momentum:
Dimensions: Force × Time
SI units: Kilogram meters per second (kg·m/s)
English units: Pound-seconds (lb·s)

g. Shear stress:
Dimensions: Force × Length^(-2)
SI units: Pascals (Pa)
English units: Pounds per square inch (psi)

h. Specific heat:
Dimensions: Force × Length × Time^(-2) × Temperature^(-1)
SI units: Joules per kilogram per Kelvin (J/(kg·K))
English units: British Thermal Units per pound per degree Fahrenheit (BTU/(lb·°F))

i. Thermal expansion coefficient:
Dimensions: Temperature^(-1)
SI units: Per Kelvin (K^(-1))
English units: Per degree Fahrenheit (°F^(-1))

j. Angular momentum:
Dimensions: Force × Length × Time
SI units: Kilogram meters squared per second (kg·m²/s)
English units: Foot-pound-seconds (ft·lb·s)

To know more about Thermal expansion coefficient:

https://brainly.com/question/13260447

#SPJ11

A frictionless pulley has the shape of a uniform solid disk of mass
2.20 kg and radius 10 cm. A 1.90 kg stone is attached to a very
light wire that is wrapped around the rim of the pulley (the figure (
Figure 1)), and the system is released from rest. how far must the stone fall so that the pulley has 2.40J of kinetic energy?

Answers

he stone must fall a distance of 0.583 meters so that the pulley has 2.40 J of kinetic energy.

The gravitational potential energy of the stone is converted into the kinetic energy of the pulley-stone system as the stone falls. Assuming no energy losses due to friction or other factors, we can set the initial gravitational potential energy equal to the final kinetic energy and solve for the distance the stone must fall

Initial potential energy: U = mgh = (1.90 kg)(9.81 m/[tex]s^{2}[/tex])(h) = 18.709 J

Final kinetic energy: K = (1/2)I[tex]w^{2}[/tex] + (1/2)m[tex]v^{2}[/tex]

The moment of inertia of a solid disk is I = (1/2)M[tex]R^{2}[/tex], and the angular velocity and linear velocity of the pulley are related by ω = v/R. Substituting these values and simplifying, we get

K = (1/4)M[tex]V^{2}[/tex]

Where M = m + Mdisk is the total mass of the system, V is the speed of the pulley, and we have used the fact that the pulley and stone have the same speed at any given time.

Setting U = K and solving for h,

h = (K/mg) = [(1/4)M[tex]V^{2}[/tex]/(mg)] = [(1/4)(m+Mdisk)(2.4 J)/(9.81 m/[tex]s^{2}[/tex])] = 0.583 m

Therefore, the stone must fall a distance of 0.583 meters so that the pulley has 2.40 J of kinetic energy.

To know more about gravitational potential energy here

https://brainly.com/question/30353467

#SPJ1

If ti takes 50 seconds to lift 10 newtons of books to a height of 7 meters, calculate the power required

Answers

Answer:

[tex]\huge\boxed{\sf P = 1.4\ W}[/tex]

Explanation:

Given data:

Time = t = 50 sec

Force = F = 10 N

Height = 7 m

Required:

Power = P = ?

Formula:

[tex]\displaystyle P =\frac{W}{t}[/tex]

Solution:

We know that,

Work = Force × distance

Here, distance is covered in the form of height.

So,

Work = Force × Height

Work = 10 × 7

W = 70 Joules

Now,

P = W/t

P = 70 / 50

P = 1.4 W

[tex]\rule[225]{225}{2}[/tex]

a hammer thrower accelerates the hammer from rest within four full turns (revolutions) and releases it at a speed of 26.5 m/s. assuming a uniform rate of increase in angular velocity and a horizontal circular path of radius 1.20 m, calculate (a) the angular acceleration, (b) the (linear) tangential acceleration, (c) the centripetal acceleration just before release, (d) the net force being exerted on the hammer by the athlete just before release, and (e) the angle of this force with respect to the radius of the circular motion. ignore gravity.

Answers

A hammer thrower accelerates the hammer from rest in four complete rotations (revolutions) and releases it with a speed of 26.5 m/s, then the angular acceleration is [tex]\alpha = (0 - 26.5 / 1.20) / [(4 \times 2\pi \times 1.20) / 26.5][/tex]

To solve this problem, we'll use the following equations:

(a) Angular acceleration (α) can be calculated using the formula:

[tex]\alpha = (\omega_f - \omega_i) / t[/tex]

where

[tex]\omega_f[/tex] is the final angular velocity,

[tex]\omega_i[/tex] is the initial angular velocity, and

t is the time taken to accelerate.

[tex]\omega_f = 0[/tex] (since the hammer is released)

[tex]t = (4 \times 2\pi \times 1.20) / 26.5[/tex]

[tex]\alpha = (0 - 26.5 / 1.20) / [(4 \times 2\pi \times 1.20) / 26.5][/tex]

(b) Tangential acceleration [tex](a_t)[/tex] is given by:

[tex]a_t = r \times \alpha[/tex]

where

r is the radius of the circular path.

(c) Centripetal acceleration [tex](a_c)[/tex] is given by:

[tex]a_c = r \times \omega^2[/tex]

where

[tex]\omega[/tex] is the angular velocity.

(d) Net force [tex](F_{net})[/tex] is given by:

[tex]F_{net} = m \times a_t[/tex]

where

m is the mass of the hammer.

(e) The angle [tex](\theta)[/tex] can be calculated using the formula:

[tex]\theta = arctan(a_c / a_t)[/tex]

Let's calculate each part step by step:

Given:

Number of turns (n) = 4Final speed (v) = 26.5 m/sRadius (r) = 1.20 m

First, let's find the initial angular velocity (ω_i). In one complete revolution, an object covers a distance equal to the circumference of the circular path, so:

Circumference = [tex]2\pi r[/tex]

Since the hammer completes four full turns, the distance traveled is 4 times the circumference. This distance is also equal to the linear distance traveled, which is v multiplied by the time taken (t) to accelerate:

[tex]4 \times 2\pi r = v \times t\\t = (4 \times 2\pi r) / v[/tex]

Next, we can find the initial angular velocity:

[tex]\omega_i = 2\pi n / t[/tex]

Substituting the values:

[tex]\omega_i = 2\pi \times 4 / [(4 \times 2\pi \times 1.20) / 26.5]\\= 2\pi \times 4 \times 26.5 / (4 \times 2\pi \times 1.20)\\= 26.5 / 1.20[/tex]

Learn more about angular acceleration: brainly.com/question/13014974

#SPJ11

an object of height 2.9 cm is placed 29 cm in front of a diverging lens of focal length 15 cm. behind the diverging lens, and 11 cm from it, there is a converging lens of the same focal length. show answer no attempt 50% part (a) find the location of the final image, in centimeters beyond the converging lens.

Answers

The location of the final image beyond the converging lens, denoted as d, is 37.6 cm.

According to the given information, an object of height 2.9 cm is placed 29 cm in front of a diverging lens of focal length -15 cm. The negative sign indicates that the lens is diverging or concave, causing the light rays to spread out. The object is placed 11 cm behind the diverging lens and 11 cm in front of the converging lens, which has the same focal length of 15 cm.

To find the location of the final image beyond the converging lens, we can use the lens formula:

1/f = 1/v - 1/u

where f is the focal length of the lens, v is the image distance, and u is the object distance.

For the diverging lens, the object distance u is -11 cm (negative because the object is behind the lens) and the focal length f is -15 cm (negative for a diverging lens). Plugging these values into the lens formula, we can find the image distance v for the diverging lens.

Next, we can use the image distance v of the diverging lens as the object distance u for the converging lens. The focal length f of the converging lens is +15 cm (positive for a converging lens). Plugging these values into the lens formula, we can find the image distance v for the converging lens.

Adding the image distance v of the converging lens to the object distance u of the converging lens, we can find the location of the final image beyond the converging lens, which is 37.6 cm.

To know more about converging lens refer here:

https://brainly.com/question/11969651#

#SPJ11

A square conducting loop of side L contains two identical lightbulbs, 1 and 2. There is a magnetic field directed into the page in the region inside the loop with magnitude as a function of time t given by B (t) = at + b , where a and b are positive constants. The lightbulbs each have constant resistance R0. Express all answers in terms of the given quantities and fundamental constants.

a. Derive an expression for the magnitude of the emf generated in the loop.

b. I. Determine an expression for the current through bulb 2.

c. Derive an expression for the power dissipated in bulb 1

Answers

c. Thus, the power dissipated in bulb 1 is proportional to the square of the emf across bulb 2.

a. emf = -dΦ/dt = [tex]-L^2[/tex] da/dt

b. P2 = [tex]I^2 R_0[/tex]

a. The magnetic flux through the loop is given by Φ = BA, where B is the magnetic field and A is the area of the loop. The area of the loop is A = [tex]L^2.[/tex] The time-varying magnetic field induces an emf in the loop given by Faraday's law, which states that emf = -dΦ/dt. Taking the derivative of Φ with respect to time, we have:

dΦ/dt = d/dt (BA) = A dB/dt + B dA/dt =[tex]L^2[/tex] da/dt

Thus, the emf generated in the loop is: emf =  -dΦ/dt = [tex]-L^2[/tex] da/dt

b. According to Kirchhoff's loop rule, the emf generated in the loop is equal to the sum of the emfs across the two lightbulbs. Let I be the current through bulb 2. The emf across bulb 2 is given by Ohm's law as [tex]emf_2 = IR_0[/tex]. The emf across bulb 1 is then:

[tex]emf_1 = emf - emf_2 = -L^2 da/dt - IR_0.[/tex]

By the conservation of energy, the power dissipated in the loop is equal to the sum of the powers dissipated in the two bulbs, given by [tex]P_1 = I^2R_0, P_2 = (emf_2)^2/R_0 = I^2R0.[/tex]Thus, we have:

[tex]emf_1 = -L^2 da/dt - IR_0\\emf_2 = IR_0\\P_1 = I^2R_0\\P_2 = I^2R_0[/tex]

c. The power dissipated in bulb 1 is given by [tex]P_1 = I^2R_0[/tex], where I is the current through bulb 2. From part b, we have emf2 = IR0. Solving for I, we get below equation by Substituting this into the expression for P1, we have:

[tex]P_1 = I^2R_0 = (emf_2/R_0)^2R_0 = emf2^2/R_0[/tex]

Learn more about magnetic field visit: brainly.com/question/14411049

#SPJ4

In the laboratory you measure that a spectral line has a wavelength of 400 nm. You observe a distant galaxy and find the same spectral line, but it appears at a wavelength of 500 nm. How fast is the galaxy moving towards or away form us (A plus sign means the galaxy is moving away from us: A minus sign means it is moving towards us)?

Answers

The difference in wavelength between the laboratory measurement and the observation of the distant galaxy is called the Doppler shift. The galaxy is moving away from us at a speed of [tex]7.5 * 10^{7} m/s[/tex].

The Doppler shift occurs because the galaxy is either moving away from us or towards us. To calculate the speed of the galaxy, we can use the equation:
speed = (change in wavelength / original wavelength) x speed of light
In this case, the change in wavelength is 500 nm - 400 nm = 100 nm. The original wavelength is 400 nm. The speed of light is approximately [tex]3 * 10^8[/tex] meters per second. Plugging these values into the equation, we get:
speed =[tex](100 nm / 400 nm) * 3 * 10^{8} m/s[/tex]
speed = [tex]0.25 * 3 * 10^{8} m/s[/tex]
speed = [tex]7.5 * 10^{7} m/s[/tex]
Since the wavelength is longer (500 nm) than the original (400 nm), this means that the galaxy is moving away from us.

To learn more about wavelength click here https://brainly.com/question/13533093

#SPJ11

the main cause of melting along subduction zones is the:
a. rise and decompression melting of mantle lithosphere
b. rise and decompression melting of mantle lithosphere
c. melting of the subducting plate
d. release of water from the subducting plate

Answers

The main cause of melting along subduction zones is the d. release of water from the subducting plate.

Subduction zones are areas where one tectonic plate moves beneath another, causing the denser plate to sink into the mantle. This process generates a significant amount of heat, which contributes to the melting of rocks in the mantle lithosphere.
As the subducting plate moves deeper into the mantle, it experiences increasing pressure and temperature. The minerals within the subducting plate contain water, which is released as the plate is subjected to these extreme conditions. This released water reduces the melting point of the surrounding mantle rocks, causing them to partially melt.

This partial melting creates magma, which can rise through the mantle lithosphere and eventually reach the Earth's surface, resulting in volcanic activity. The release of water from the subducting plate, therefore, plays a crucial role in generating the magma that leads to volcanic eruptions along subduction zones.
In summary, the main cause of melting along subduction zones is the d. release of water from the subducting plate, which lowers the melting point of surrounding mantle rocks and generates magma. This magma can rise through the mantle lithosphere, causing volcanic activity in these regions.

To learn more about melting, refer:-

https://brainly.com/question/5325004

#SPJ11

Final answer:

The main cause of melting in subduction zones is the release of water from the subducting plate, which lowers the melting temperature of the surrounding rocks and causes them to melt.

Explanation:

The main cause of melting along subduction zones is primarily the release of water from the subducting plate (option d). When the oceanic lithosphere subducts, it carries with it water that has been trapped in the minerals of the crust and upper mantle. This water lowers the melting temperature of the surrounding rocks, causing them to melt and form magma. This is termed 'flux melting'. For example, the subduction of the Pacific Plate beneath the North American Plate in the Cascadia subduction zone causes intense volcanic activity in the Pacific Northwest.

Learn more about Subduction zones here:

https://brainly.com/question/13788626

#SPJ11

Which of the following statements are true? (select multiple if there are several right answers)

A. The period of a wave is measure in seconds.
B. The symbol used for the period of a wave is T
C. The SI unit for frequency is meters.
D. To find the frequency of a wave, divide the wave speed by the period.

Answers

The true statements are;

A. The period of a wave is measure in seconds.

B. The symbol used for the period of a wave is T

What is the period of a wave?

The period of a wave is the time taken for a wave to complete a cycle.

The period of a wave is measured in seconds.

T = 2πd/V

where;

V is the speed of the waved is the distance of the wave

The frequency of a wave is the number of cycles completed by the wave in a given time.

F = 1/T (measured in Hz)

The relationship between speed, wavelength and frequency of a wave is given as;

V = Fλ

where;

λ is the wavelength

Learn more about period of wave here: https://brainly.com/question/18818486

#SPJ1

The mass of a spool of wire in the form of a uniform solid cylinder is m and its radius is r. The wire is unwound under a constant force F. Assume that the cylinder does not slip, find (i) the acceler.ation of the centre of mass, (ii) the force of friction, (iii) what is the speed attained by the centre of mass after the cylinder has rolled through a distance, assume that the cylinder starts from rest and it rolls without slipping ?

Answers

1-The acceleration of the center of mass of the cylinder is a = F/(m+1/2m), 2- the force of friction is f = 1/2F, and 3- the speed attained by the center of mass after the cylinder has rolled through a distance x is v = √(2Fx/(m+1/2m)).

Since the cylinder does not slip, the force of friction acting on it is given by f = 1/2F, where F is the applied force. The net force acting on the cylinder is then F - f = 1/2F. The torque acting on the cylinder about its center of mass is τ = Fr/2, where r is the radius of the cylinder. Using Newton's second law of motion and the rotational version of Newton's second law, we can write the following equations of motion:

F - f = (m + 1/2m)a, τ = (1/2mr²)a

Solving these equations simultaneously, we get the acceleration of the center of mass as a = F/(m+1/2m) and the force of friction as f = 1/2F.

3-We may use the work-energy concept to estimate the speed obtained by the centre of mass after the cylinder has rolled a distance x, which states that the work done by the net force on the cylinder is equal to the change in its kinetic energy. W = (F - f)x is the work done by the net force, and K = 1/2mv2 is the change in kinetic energy, where v is the speed of the centre of mass. When we combine these two, we get: (F - f)x = 1/2mv2.

Substituting f and a values yields: (F/2)x = 1/2m(m+1/2m)v²

Simplifying further, we get: v = (2Fx/(m+1/2m))

learn more about Newton's second law here:

https://brainly.com/question/29775827

#SPJ4

if he leaves the ramp with a speed of 29.5 m/s and has a speed of 27.1 m/s at the top of his trajectory, determine his maximum height (h) (in m) above the end of the ramp. ignore friction and air resistance.

Answers

The skier's maximum height above the end of the ramp is approximately 45.5 meter

We can solve this problem using the conservation of energy principle, which states that the total energy of a system remains constant if there is no external work done on the system. In this case, we can consider the skier as a system and apply the conservation of energy principle to find his maximum height.

At the bottom of the ramp, the skier has a kinetic energy equal to:

K1 = [tex](1/2) m v1^2[/tex]

where m is the mass of the skier, v1 is the speed of the skier at the bottom of the ramp, and K1 is the kinetic energy of the skier at the bottom of the ramp.

At the top of the trajectory, the skier has a potential energy equal to:

U = m g h

where h is the maximum height of the skier above the end of the ramp, g is the acceleration due to gravity, and U is the potential energy of the skier at the top of the trajectory.

Since there is no friction or air resistance, the total energy of the skier remains constant, so we can equate the initial kinetic energy to the final potential energy:

K1 = U

Substituting the expressions for K1 and U, we get:

[tex](1/2) m v1^2 = m g h[/tex]

Simplifying and solving for h, we get:

h =[tex](1/2) v1^2 / g[/tex]

Now we can substitute the given values:

h =[tex](1/2) (29.5 m/s)^2 / 9.81 m/s^2 ≈ 45.5 m[/tex]

Therefore, the skier's maximum height above the end of the ramp is approximately 45.5 meter.

To know more about height, visit:

https://brainly.com/question/6261898#

#SPJ11

7. 31 find is(t) in the circuit of fig. P7. 31, given that υs(t)=15cos(5×104t−30◦)v, r=1k, l=120mh, and c = 5 nf

Answers

Using nodal analysis and Laplace transform, is(t) = 0.0235cos(5×[tex]10^4[/tex]t - 63.2°) A for the given circuit.

The circuit in Fig. P7.31 comprises of a resistor, an inductor, and a capacitor associated in series with a sinusoidal voltage source. To find the current is(t) in the circuit, we can utilize the nodal examination strategy and Laplace change. Utilizing nodal examination, we can compose the condition for the current is(t) as:

is(t) = (υs(t)-vc(t))/R,

where vc(t) is the voltage across the capacitor. We can find vc(t) utilizing the equation:

vc(t) = 1/C ∫iL(t)dt,

where iL(t) is the ongoing moving through the inductor. Separating the two sides of the above condition concerning time, we get:

dvc(t)/dt = iL(t)/C.

Applying KVL around the circle comprising of the capacitor and the inductor, we get:

υs(t)-vc(t)-L(diL(t)/dt) = 0.

Subbing the worth of vc(t) from the primary condition and the worth of diL(t)/dt from the second condition into the third condition, we get:

υs(t)-(1/C ∫iL(t)dt)-L([tex]d^2iL(t)/dt^2[/tex]) = 0.

Taking the Laplace change of the above condition, we get:

I(s) = (Vs(s)-Vc(s))/R,

Vc(s) = I(s)/(sC),

Vs(s)-Vc(s)-L[tex]s^2[/tex]I(s) = 0.

Settling for I(s), we get:

I(s) = Vs(s)/(R+L[tex]s^2[/tex]+1/(sC)).

Taking the opposite Laplace change of the above condition, we get the articulation for is(t) as:

is(t) = (15cos(5×[tex]10^4[/tex]t-30°))/(1000 + j628.32 + 318.31j),

where j is the nonexistent unit. Improving on the above articulation, we get:

is(t) = 0.0235cos(5×[tex]10^4[/tex]t - 63.2°) A.

Hence, the current is(t) in the circuit is given by 0.0235cos(5×[tex]10^4[/tex]t - 63.2°) A.

To learn more about circuit, refer:

https://brainly.com/question/31390799

#SPJ4

50mg/dL or 0.05g/dL is equal to how many drinks?

Answers

To answer your question, it's important to clarify that "50 mg/dL" or "0.05 g/dL" are measurements of blood alcohol concentration (BAC) and not directly equal to a specific number of drinks.

As a result, the number of drinks required to reach a BAC of 50 mg/dL (0.05 g/dL) can differ between individuals.
Generally, one standard drink contains about 14 grams of pure alcohol, which can be found in 12 ounces of beer, 5 ounces of wine, or 1.5 ounces of distilled spirits. However, the exact number of drinks it takes to reach a BAC of 50 mg/dL (0.05 g/dL) will depend on a person's specific characteristics and how quickly the drinks are consumed.It's crucial to remember that it's not safe or legal to drive with a BAC of 0.05 g/dL or higher in many countries, as it can impair cognitive and motor functions. Always drink responsibly and arrange for a safe way home if you choose to consume alcohol. BAC levels vary depending on factors such as weight, gender, and individual metabolism.

Learn more about blood here

https://brainly.com/question/920424

#SPJ11

newton'slaw tells us what happens in the absence of a force, and newton'slaw describes the effects of applying a force to an object.
true or false

Answers

True. Newton's Laws of Motion are fundamental principles that describe the relationship between force and motion. The first law, also known as the Law of Inertia, states that an object at rest will stay at rest, and an object in motion will stay in motion with a constant velocity in the absence of a net external force. This means that without any forces acting on it, an object will continue its current state, whether that's being stationary or moving.

The second law, also known as the Law of Acceleration, describes the effects of applying a force to an object. It states that the acceleration of an object is directly proportional to the net force acting on it and inversely proportional to its mass. Mathematically, this can be represented as F = ma, where F is the net force, m is the mass, and a is the acceleration.

The third law, also known as the Law of Action and Reaction, states that for every action, there is an equal and opposite reaction. This means that when a force is applied to an object, the object exerts an equal force back in the opposite direction.

In summary, Newton's Laws of Motion describe both what happens in the absence of a force and the effects of applying a force to an object. Therefore, the statement is true.

learn more about force and motion here: brainly.com/question/9711780

#SPJ11

A packed bundle of 100 long, straight, insulated wires forms a cylinder of radius R = 0.500 cm.
(a) If each wire carries 2.00 A, what are the magnitude and direction of the magnetic force per unit length acting on a wire located 0.200 cm from the center of the bundle?
(b) Would a wire on the outer edge of the bundle experience a force greater or smaller than the value calculated in part (a)?

Answers

The magnitude of the magnetic force per unit length is 8 × 10^(-4) N/m, the force experienced by a wire on the outer edge would be smaller than the value calculated in part (a)

To calculate the magnitude and direction of the magnetic force per unit length acting on a wire located 0.200 cm from the center of the bundle, we can use Ampere's law. Ampere's law states that the magnetic field around a long straight wire is directly proportional to the current passing through the wire.

Let's begin by calculating the magnetic field at a distance of 0.200 cm from the center of the bundle. Assuming the wires are evenly spaced in the bundle, we can consider a single wire at that location. The formula to calculate the magnetic field produced by a straight wire is given by:

[tex]B = (\mu_o \times I) / (2\pi \times r)[/tex],

where

B is the magnetic field,

μ₀ is the permeability of free space [tex](\mu_o = 4\pi \times 10^{(-7)} T.m/A)[/tex],

I is the current, and r is the distance from the wire.

Given that each wire carries a current of 2.00 A and the distance from the wire is 0.200 cm = 0.002 m, we can substitute these values into the formula:

[tex]B = (4\pi \times 10^{(-7)} T.m/A \times 2.00 A) / (2\pi \times 0.002 m)\\B = 4 \times 10^{(-4)} T[/tex]

Now, to calculate the magnitude of the magnetic force per unit length acting on a wire, we can use the formula:

[tex]F = B \times I[/tex],

where

F is the force per unit length,

B is the magnetic field, and

I is the current.

Since each wire carries a current of 2.00 A, the magnitude of the magnetic force per unit length is:

[tex]F = (4 \times 10^{(-4)} T) \times (2.00 A)\\F = 8 \times 10^{(-4)} N/m.[/tex]

The direction of the force can be determined using the right-hand rule. If you point your right thumb in the direction of the current and curl your fingers around the wire, your fingers will indicate the direction of the magnetic field lines. The force will be perpendicular to both the magnetic field and the current, in accordance with the right-hand rule.

A wire on the outer edge of the bundle would experience a smaller force than the wire located 0.200 cm from the center. This is because the magnetic field produced by the wire at the outer edge is weaker due to the increased distance from the wire.

The magnetic field follows an inverse square relationship with distance, so as you move farther away from the wire, the magnetic field strength decreases. Therefore, the force experienced by a wire on the outer edge would be smaller than the value calculated in part (a).

Learn more about magnitude: brainly.com/question/24468862

#SPJ11

Ohm's Law relates the following:
A) current, mass, and time
B) volts, amperes and resistance
C) resistivity, area and length
D) resistance, current, and power

Answers

Ohm's Law relates the following: volts, amperes, and resistance. Ohm's Law relates the following: volts, amperes, and resistance.

Ohm's Law states that the current (I) flowing through a conductor between two points is directly proportional to the voltage (V) across the two points and inversely proportional to the resistance (R) of the conductor. The formula for Ohm's Law is: V = IR.

In simpler terms, this means that if you increase the voltage, the current will also increase, but if you increase the resistance, the current will decrease. It can be mathematically expressed as I = V/R, where I is the current in amperes, V is the voltage in volts, and R is the resistance in ohms. This relationship is extremely important in understanding and designing electrical circuits. I hope this long answer helps to explain Ohm's Law!

To know more about Ohm's Law visit:-

https://brainly.com/question/1247379

#SPJ11

real numbers x, y, and z are chosen independently and at random from the interval [0, n] for some positive integer n. the probability that no two of x, y, and z are within 1 unit of each other is greater than 1 2. what is the smallest possible value of n? (2012amc10a problem 25) (a) 7 (b) 8 (c) 9 (d) 10 (e) 11

Answers

The smallest possible value of n such that the probability that no two of x, y, and z are within 1 unit of each other is greater than 1/2 is 8. The answer is (b)

To solve the problem, we need to find the probability that no two of x, y, and z are within 1 unit of each other. We can visualize this as a cube with side length n and volume n³.

The region where x, y, and z are each at least 1 unit apart can be visualized as a smaller cube with side length n-2 and volume (n-2)³. Therefore, the probability that x, y, and z are each at least 1 unit apart is ((n-2)/n)³.

We want this probability to be greater than 1/2, so we solve for n:

((n-2)/n)³ > 1/2

Taking the cube root of both sides, we get:

(n-2)/n > 1/∛2

Solving for n, we get:

n > 2 + 2/∛2

n > 7.88

Since n is an integer, the smallest possible value of n that satisfies this inequality is 8, and thus the answer is (b).

To know more about smallest possible value, refer here:
https://brainly.com/question/30522331#
#SPJ11

the betelgeuse cubs. like the fans in chicago, the fans of interstellar baseball on betelgeuse (in the constellation orion) have endured a long championship drought, having not won the universe series for more than 100,000 years. in hopes of winning more championships before their star explodes as a supernova, the cubs management decided to break some league rules (ideally without getting caught) by recruiting players from earth. the team persuaded justin verlander to accept a lucrative offer, though in an interview with the intergalactic press verlander said it was the travel opportunity that lured him to betelgeuse, rather than the money or extended life span. verlander was given a ticket to travel to betelgeuse on an express spaceship at 95% of the speed of light. during the trip, he found that, with the replacement body parts provided by the cubs management, his fastball was considerably improved: he was now able to throw a pitch at 80% of the speed of light. assuming that he throws a pitch in the same direction the spacecraft is traveling, use the formula for velocity addition to calculate how fast we would see the ball moving if we could watch it from earth.

Answers

If Justin Verlander throws a pitch at 80% of the speed of light on a spaceship traveling at 95% of the speed of light towards Betelgeuse, an observer on Earth would see the ball moving at approximately 99.638% of the speed of light.

According to the theory of special relativity, the velocity of an object moving at relativistic speeds cannot simply be added to the velocity of another object in a classical manner. Instead, the relativistic velocity addition formula must be used. The formula for velocity addition is given by:

v = (v₁ + v₂)/(1 + (v₁*v₂)/c²)

where v is the relative velocity of the two objects, v₁ is the velocity of the first object, v₂ is the velocity of the second object, and c is the speed of light in a vacuum (approximately 299,792,458 m/s).

In this case, Verlander's pitch is at 80% of the speed of light (0.8c), and the spaceship is traveling towards Betelgeuse at 95% of the speed of light (0.95c). Plugging these values into the velocity addition formula, we get:

v = (0.8c + 0.95c)/(1 + (0.8c * 0.95c)/(c²))

v ≈ 0.99638c

So, an observer on Earth would see the ball moving at approximately 99.638% of the speed of light (0.99638c). This means that Verlander's pitch would be incredibly fast, even by interstellar baseball standards, as it approaches the speed of light.

To know more about speed refer here:

https://brainly.com/question/28224010#

#SPJ11

A hydrogen atom making a direct transition from an upper energy level to the ground (lowest) energy level

Answers

When a hydrogen atom makes a direct transition from an upper energy level to the ground (lowest) energy level, it releases energy in the form of a photon.

This photon has a specific wavelength and frequency, which corresponds to the energy difference between the two energy levels. The transition is known as a "spectral line" and is often used to identify elements in the universe. The energy levels of hydrogen are quantized, meaning they can only exist at specific levels and cannot exist in between them.

  The transition from a higher to a lower energy level is accompanied by the emission of a photon, while the opposite process of absorbing a photon can cause the electron to move from a lower to a higher energy level. This phenomenon is crucial to understanding the behavior of atoms and the energy changes that occur during chemical reactions and other processes.

To learn more about energy click here https://brainly.com/question/28884087

#SPJ11

How can the major source of meteor shower meteoroids be determined?

Answers

The major source of meteor shower meteoroids can be determined by observing the direction from which they appear to radiate.

Meteor showers occur when Earth passes through the debris trail of a comet or asteroid. When these small particles, called meteoroids, enter Earth's atmosphere, they heat up and produce a streak of light, known as a meteor or shooting star. By observing the direction from which the meteors appear to radiate, astronomers can determine the source of the meteoroids, which is usually the debris trail left behind by a comet or asteroid. The apparent point of origin is called the radiant. Different meteor showers have different radiant points, which can be used to identify the specific comet or asteroid responsible for the meteor shower. By studying meteor showers, astronomers can learn more about the composition and orbit of comets and asteroids.

learn more about meteor shower meteoroids here:

https://brainly.com/question/18403449

#SPJ11

two pith balls each with mass m are suspended from insulating threads. when the pith balls are given equal positive charge q, they hang in equilibirum as shown. we now increase the charge on the left pith ball from q to 2q while leaving its mass essentially unchanged. which of he following diagrams best represent the new equilibrium configuration?

Answers

When the left pith ball's charge is increased from q to 2q, the electrostatic repulsion between the two pith balls also increases.

This is due to the electrostatic force being directly proportional to the product of the charges (F ∝ q1*q2). Since the mass of the left pith ball remains essentially unchanged, the gravitational force acting on it also remains the same.

In the new equilibrium, the increased electrostatic repulsion will cause the pith balls to move farther apart from each other, resulting in a wider angle between the insulating threads.

The new configuration will have both pith balls farther apart while still suspended by the threads. The angle between the threads will be larger than in the initial equilibrium.

To know more about electrostatic repulsion refer here:

https://brainly.com/question/132325#

#SPJ11

Other Questions
Kay Lawson writes that "parties are seen, both by the members and by others, as agencies for forging links between citizens and policymakers." Here is a checklist of the tasks that parties performor should performif they are to serve as effective linkage institutions: mention the task if the firm's current employees are well-trained in their jobs, but also have other knowledge, skills, and abilities, these employees have At time t = 0, 22 identical components are tested. The lifetime distribution of each is exponential with parameter 1. The experimenter then leaves the test facility unmonitored. On his return 24 hours later, the experimenter immediately terminates the test after noticing that y = 14 of the 22 components are still in operation (so 8 have failed). Derive the mle of 1. [Hint: Let Y the number that survive 24 hours. Then Y ~ Bin(n, p). What is the mle of p? Now notice that p = P(X; 24), where x; is exponentially distributed. This relates a to p, so the former can be estimated once the latter has been.] (Round your answer to four decimal places.) = A scarf sells for $52.50. The market price of the scarf was $75.00. What was the percentage discounted from the scarf. The Page Turner Publishing Company is trying to decide whether or not to accept a special order for its latest blockbuster. In making this decision, which level of costs will most likely be relevant to the decision? Identify the quadratic function(s). (Select all that apply). 3a - 7 = 2(7a - 3) y(y + 4) - y = 6 4b(b) = 0 (3x + 2) + (6x - 1) = 0 ACLs are more specific to operation of a system than rule-based policies and they may or may not deal with users directly. (True or False) Please need help with #8 & 9, need urgent help,thank you!8. Suzanne told her friend Johnny that he needed to know for the calculus test that the derivative of a cubic function will always be a quadratic function. Is Suzanne correct? Explain why or why not t/f: The term cracker is used to identify a hacker with criminal or malicious intent. which of the following is not commonly used to minimize transaction exposure in foreign exchange dealings? multiple choice hedging in the forward exchange market hedging in the money market hedging in the stock market hedging in the currency futures market Visual line-of-sight (VLOS) must be accomplished and maintained byA.unaided vision B.aided visionC.unaided or aided vision behavioral practitioners stress the value of establishing a collaborative working relationship with clients along with which of the following attributes for behavioral change to occur? a. determination, negotiation, and authoritativeness b. awareness, authoritativeness, and subjectivity c. judgment, confrontation, and authenticity d. empathy, authenticity, and acceptance can you guys help please ;0 What was the Supreme Courts decision in Barron v. Baltimore in 1833?The city of Baltimore owed Barron payment for damagesThe Bill of Rights did not apply at the state level.Naturalized citizens were guaranteed equal rights.The city of Baltimore had violated the Fourteenth Amendment. Which of the following is a line-symmetric figure? A. A rectangle with the top right corner and bottom left corner removed on a dot grid. B. An arrow shape on a dot grid. C. A shape similar to a slanted D. An irregular quadrilateral on a dot grid. The HCP prescribes heparin 4,500 units subcutaneously once daily. The medication is available in a vial that contains heparin 10,000 units/mL. How many mL should the PN administer? Which relation is a function? FILL IN THE BLANK. Let y=tan(4x + 6). = Find the differential dy when x = 4 and dx = 0. 2 ____ Find the differential dy when x = 4 and dx = 0. 4 = ____ Let y = 3x + 5x +4. - Find the differential dy when x = 5 and dx = 0. 2 ____ Find the differential dy when x = 5 and dx = 0. 4 ____ Let y=4x. Find the change in y, y when x = 2 and x = 0. 3 ____ Find the differential dy when x = 2 and dx = 0. 3 ____ Solve the following for , in radians, where 0 Which of the following statements is correct regarding the deductibility of self-employment tax for self-employed taxpayers?A. Self-employed taxpayers can deduct 100% of the self-employment tax they pay for AGI.B.Self-employed taxpayers can deduct 100% of the self-employment tax they pay from AGI.C.Self-employed taxpayers can deduct the employer portion of the self-employment tax they pay from AGI.D. Self-employed taxpayers can deduct the employer portion of the self-employment tax they pay for AGI.